Difference between revisions of "2024 AIME II Problems/Problem 12"

(Solution 1 (completely no calculus required))
 
(26 intermediate revisions by 14 users not shown)
Line 1: Line 1:
Let <math>O(0,0),A(\tfrac{1}{2},0),</math> and <math>B(0,\tfrac{\sqrt{3}}{2})</math> be points in the coordinate plane. Let <math>\mathcal{F}</math> be the family of segments <math>\overline{PQ}</math> of unit length lying in the first quadrant with <math>P</math> on the <math>x</math>-axis and <math>Q</math> on the <math>y</math>-axis. There is a unique point <math>C</math> on <math>\overline{AB},</math> distinct from <math>A</math> and <math>B,</math> that does not belong to any segment from <math>\mathcal{F}</math> other than <math>\overline{AB}</math>. Then <math>OC^2=\tfrac{p}{q}</math>, where <math>p</math> and <math>q</math> are relatively prime positive integers. Find <math>p+q</math>.
+
==Problem==
 +
Let \(O=(0,0)\), \(A=\left(\tfrac{1}{2},0\right)\), and \(B=\left(0,\tfrac{\sqrt{3}}{2}\right)\) be points in the coordinate plane. Let \(\mathcal{F}\) be the family of segments \(\overline{PQ}\) of unit length lying in the first quadrant with \(P\) on the \(x\)-axis and \(Q\) on the \(y\)-axis. There is a unique point \(C\) on \(\overline{AB}\), distinct from \(A\) and \(B\),  that does not belong to any segment from \(\mathcal{F}\) other than \(\overline{AB}\). Then \(OC^2=\tfrac{p}{q}\), where \(p\) and \(q\) are relatively prime positive integers. Find \(p+q\).
 +
 
 +
 
 +
==Solution 1 (completely no calculus required)==
 +
 
 +
Begin by finding the equation of the line <math>\overline{AB}</math>: <math>y= -\sqrt{3}x+\frac{\sqrt{3}}{2}</math> Now, consider the general equation of all lines that belong to <math>\mathcal{F}</math>. Let <math>P</math> be located at <math>(a,0)</math> and <math>Q</math> be located at <math>(0,b)</math>. With these assumptions, we may arrive at the equation <math>ay +bx =ab</math>. However, a critical condition that must be satisfied by our parameters is that <math>a^2+b^2=1</math>, since the length of <math>\overline{PQ}=1</math>.
 +
 
 +
Here's the golden trick that resolves the problem: we wish to find some point <math>C</math> along <math>\overline{AB}</math> such that <math>\overline{PQ}</math> passes through <math>C</math> iff <math>a=\frac{1}{2}</math>. It's not hard to convince oneself of this, since the property <math>a^2+b^2=1</math> implies that if <math>a=\frac{1}{2}</math>, then <math>\overline{PQ}=\overline{AB}</math>.
 +
 
 +
We should now try to relate the point <math>C</math> to some value of <math>a</math>. This is accomplished by finding the intersection of two lines: <cmath>
 +
 
 +
\[
 +
    a(-\sqrt{3}x +\frac{\sqrt{3}}{2}) + x\sqrt{1-a^2} = a\sqrt{1-a^2}
 +
\]
 +
 
 +
</cmath>
 +
 
 +
Where we have also used the fact that <math>b=\sqrt{1-a^2}</math>, which follows nicely from <math>a^2+b^2 =1</math>.  <cmath>
 +
 
 +
\[
 +
    a(-\sqrt{3}x +\frac{\sqrt{3}}{2})  = (a-x)\sqrt{1-a^2}
 +
\]
 +
 
 +
</cmath>
 +
 
 +
Square both sides and go through some algebraic manipulations to arrive at
 +
<cmath>
 +
 
 +
\[
 +
    -a^4 +2xa^3+(-4x^2+3x+\frac{1}{4})a^2-2xa+x^2=0
 +
\]
 +
 
 +
</cmath>
 +
 
 +
Note how <math>a=\frac{1}{2}</math> is a solution to this polynomial, and it is logically so. If we found the set of intersections consisting of line segment <math>\overline{AB}</math> with an identical copy of itself, every single point on the line (all <math>x</math> values) should satisfy the equation. Thus, we can perform polynomial division to eliminate the extraneous solution <math>a=\frac{1}{2}</math>. <cmath>
 +
 
 +
\[
 +
    -a^3 + (2x-\frac{1}{2})a^2+(-4x^2+4x)a-2x^2=0
 +
\]
 +
</cmath>
 +
 
 +
Remember our original goal. It was to find an <math>x</math> value such that <math>a=\frac{1}{2}</math> is the only valid solution. Therefore, we can actually plug in <math>a=\frac{1}{2}</math> back into the equation to look for values of <math>x</math> such that the relation is satisfied, then eliminate undesirable answers.
 +
<cmath>
 +
 
 +
\[
 +
    16x^2-10x+1=0
 +
\]
 +
</cmath>
 +
This is easily factored, allowing us to determine that <math>x=\frac{1}{8},\frac{1}{2}</math>. The latter root is not our answer, since on line <math>\overline{AB}</math>, <math>y(\frac{1}{2})=0</math>, the horizontal line segment running from <math>(0,0)</math> to <math>(1,0)</math> covers that point. From this, we see that <math>x=\frac{1}{8}</math> is the only possible candidate.
 +
 
 +
Going back to line <math>\overline{AB}, y= -\sqrt{3}x+\frac{\sqrt{3}}{2}</math>, plugging in <math>x=\frac{1}{8} </math> yields <math>y=\frac{3\sqrt{3}}{8}</math>. The distance from the origin is then given by <math>\sqrt{\frac{1}{8^2}+(\frac{3\sqrt{3}}{8})^2} =\sqrt{\frac{7}{16}}</math>. That number squared is <math>\frac{7}{16}</math>, so the answer is <math>\boxed{023}</math>.
 +
 
 +
 
 +
 
 +
~Installhelp_hex
 +
 
 +
==Solution 2==
 +
 
 +
<math>y=-(\tan \theta) x+\sin \theta=-\sqrt{3}x+\frac{\sqrt{3}}{2}, x=\frac{\sqrt{3}-2\sin \theta}{2\sqrt{3}-2\tan \theta}</math>
 +
 
 +
Now, we want to find <math>\lim_{\theta\to\frac{\pi}{3}}\frac{\sqrt{3}-2\sin \theta}{2\sqrt{3}-2\tan \theta}</math>. By L'Hôpital's rule, we get <math>\lim_{\theta\to\frac{\pi}{3}}\frac{\sqrt{3}-2\sin \theta}{2\sqrt{3}-2\tan \theta}=\lim_{\theta\to\frac{\pi}{3}}cos^3{x}=\frac{1}{8}</math>. This means that <math>y=\frac{3\sqrt{3}}{8}\implies OC^2=\frac{7}{16}</math>, so we get <math>\boxed{023}</math>.
 +
 
 +
~Bluesoul
 +
 
 +
==Solution 3==
 +
 
 +
The equation of line <math>AB</math> is <cmath>
 +
\[
 +
y = \frac{\sqrt{3}}{2} x - \sqrt{3} x.  \hspace{1cm} (1)
 +
\]
 +
 
 +
</cmath>
 +
 
 +
The position of line  <math>PQ</math> can be characterized by <math>\angle QPO</math>, denoted as <math>\theta</math>.
 +
Thus, the equation of line <math>PQ</math> is
 +
 
 +
<cmath>
 +
\[
 +
y = \sin \theta - \tan \theta \cdot x . \hspace{1cm} (2)
 +
\]
 +
</cmath>
 +
 
 +
Solving (1) and (2), the <math>x</math>-coordinate of the intersecting point of lines <math>AB</math> and <math>PQ</math> satisfies the following equation:
 +
 
 +
<cmath>
 +
\[
 +
\frac{\frac{\sqrt{3}}{2} - \sqrt{3} x}{\sin \theta}
 +
+ \frac{x}{\cos \theta}
 +
= 1 . \hspace{1cm} (1)
 +
\]
 +
</cmath>
 +
 
 +
We denote the L.H.S. as <math>f \left( \theta; x \right)</math>.
 +
 
 +
We observe that <math>f \left( 60^\circ ; x \right) = 1</math> for all <math>x</math>.
 +
Therefore, the point <math>C</math> that this problem asks us to find can be equivalently stated in the following way:
 +
 
 +
We interpret Equation (1) as a parameterized equation that <math>x</math> is a tuning parameter and <math>\theta</math> is a variable that shall be solved and expressed in terms of <math>x</math>.
 +
In Equation (1), there exists a unique <math>x \in \left( 0, 1 \right)</math>, denoted as <math>x_C</math> (<math>x</math>-coordinate of point <math>C</math>), such that the only solution is <math>\theta = 60^\circ</math>. For all other <math>x \in \left( 0, 1 \right) \backslash \{ x_C \}</math>, there are more than one solutions with one solution <math>\theta = 60^\circ</math> and at least another solution.
 +
 
 +
Given that function <math>f \left( \theta ; x \right)</math> is differentiable, the above condition is equivalent to the first-order-condition
 +
<cmath>
 +
\[
 +
\frac{\partial f \left( \theta ; x_C \right) }{\partial \theta} \bigg|_{\theta = 60^\circ} = 0 .
 +
\]
 +
</cmath>
 +
 
 +
Calculating derivatives in this equation, we get
 +
<cmath>
 +
\[
 +
- \left( \frac{\sqrt{3}}{2} - \sqrt{3} x_C \right) \frac{\cos 60^\circ}{\sin^2 60^\circ}
 +
+ x_C \frac{\sin 60^\circ}{\cos^2 60^\circ}
 +
= 0.
 +
\]
 +
</cmath>
 +
 
 +
By solving this equation, we get
 +
<cmath>
 +
\[
 +
x_C = \frac{1}{8} .
 +
\]
 +
</cmath>
 +
 
 +
Plugging this into Equation (1), we get the <math>y</math>-coordinate of point <math>C</math>:
 +
<cmath>
 +
\[
 +
y_C = \frac{3 \sqrt{3}}{8} .
 +
\]
 +
</cmath>
 +
 
 +
Therefore,
 +
\begin{align*}
 +
OC^2 & = x_C^2 + y_C^2 \\
 +
& = \frac{7}{16} .
 +
\end{align*}
 +
 
 +
Therefore, the answer is <math>7 + 16 = \boxed{\textbf{(23) }}</math>.
 +
 
 +
~Steven Chen (Professor Chen Education Palace, www.professorchenedu.com)
 +
 
 +
== Solution 4 (coordinate bash) ==
 +
 
 +
Let <math>s</math> be a segment in <math>\mathcal{F}</math> with x-intercept <math>a</math> and y-intercept <math>b</math>. We can write <math>s</math> as
 +
\begin{align*}
 +
\frac{x}{a} + \frac{y}{b} &= 1 \\
 +
y &= b(1 - \frac{x}{a}).
 +
\end{align*}
 +
Let the unique point in the first quadrant <math>(x, y)</math> lie on <math>s</math> and no other segment in <math>\mathcal{F}</math>. We can find <math>x</math> by solving
 +
<cmath>
 +
b(1 - \frac{x}{a}) = (b + db)(1 - \frac{x}{a + da})
 +
</cmath>
 +
and taking the limit as <math>da, db \to 0</math>. Since <math>s</math> has length <math>1</math>, <math>a^2 + b^2 = 1^2</math> by the Pythagorean theorem. Solving this for <math>db</math>, we get
 +
\begin{align*}
 +
a^2 + b^2 &= 1 \\
 +
b^2 &= 1 - a^2 \\
 +
\frac{db^2}{da} &= \frac{d(1 - a^2)}{da} \\
 +
2a\frac{db}{da} &= -2a \\
 +
db &= -\frac{a}{b}da.
 +
\end{align*}
 +
After we substitute <math>db = -\frac{a}{b}da</math>, the equation for <math>x</math> becomes
 +
<cmath>
 +
b(1 - \frac{x}{a}) = (b -\frac{a}{b} da)(1 - \frac{x}{a + da}).
 +
</cmath>
 +
 
 +
In <math>\overline{AB}</math>, <math>a = \frac{1}{2}</math> and <math>b = \frac{\sqrt{3}}{2}</math>. To find the x-coordinate of <math>C</math>, we substitute these into the equation for <math>x</math> and get
 +
\begin{align*}
 +
\frac{\sqrt{3}}{2}(1 - \frac{x}{\frac{1}{2}}) &= (\frac{\sqrt{3}}{2} - \frac{\frac{1}{2}}{\frac{\sqrt{3}}{2}} da)(1 - \frac{x}{\frac{1}{2} + da}) \\
 +
\frac{\sqrt{3}}{2}(1 - 2x) &= (\frac{\sqrt{3}}{2} - \frac{da}{\sqrt{3}})(1 - \frac{x}{\frac{1 + 2da}{2}}) \\
 +
\frac{\sqrt{3}}{2} - \sqrt{3}x &= \frac{3 - 2da}{2\sqrt{3}}(1 - \frac{2x}{1 + 2da}) \\
 +
\frac{\sqrt{3}}{2} - \sqrt{3}x &= \frac{3 - 2da}{2\sqrt{3}} \cdot \frac{1 + 2da - 2x}{1 + 2da} \\
 +
\frac{\sqrt{3}}{2} - \sqrt{3}x &= \frac{3 + 6da - 6x - 2da - 4da^2 + 4xda}{2\sqrt{3} + 4\sqrt{3}da} \\
 +
(\frac{\sqrt{3}}{2} - \sqrt{3}x)(2\sqrt{3} + 4\sqrt{3}da) &= 3 + 6da - 6x - 2da - 4da^2 + 4xda \\
 +
3 + 6da - 6x - 12xda &= 3 + 4da - 6x - 4da^2 + 4xda \\
 +
2da &= -4da^2 + 16xda \\
 +
16xda &= 2da + 4da^2 \\
 +
x &= \frac{da + 2da^2}{8da}.
 +
\end{align*}
 +
We take the limit as <math>da \to 0</math> to get
 +
<cmath>
 +
x = \lim_{da \to 0} \frac{da + 2da^2}{8da} = \lim_{da \to 0} \frac{1 + 2da}{8} = \frac{1}{8}.
 +
</cmath>
 +
We substitute <math>x = \frac{1}{8}</math> into the equation for <math>\overline{AB}</math> to find the y-coordinate of <math>C</math>:
 +
<cmath>
 +
y = b(1 - \frac{x}{a}) = \frac{\sqrt{3}}{2}(1 - \frac{\frac{1}{8}}{\frac{1}{2}}) = \frac{3\sqrt{3}}{8}.
 +
</cmath>
 +
The problem asks for
 +
<cmath>
 +
OC^2 = x^2 + y^2 = (\frac{1}{8})^2 + (\frac{3\sqrt{3}}{8})^2 = \frac{7}{16} = \frac{p}{q},
 +
</cmath>
 +
so <math>p + q = 7 + 16 = \boxed{023}</math>.
 +
 
 +
== Solution 5 (small perturb) ==
  
==Solution 1==
 
By Furaken
 
 
<asy>
 
<asy>
 
pair O=(0,0);
 
pair O=(0,0);
Line 8: Line 198:
 
pair Y=(0,1);
 
pair Y=(0,1);
 
pair A=(0.5,0); pair B=(0,sin(pi/3));
 
pair A=(0.5,0); pair B=(0,sin(pi/3));
 +
pair A1=(0.6,0); pair B1=(0,0.8);
 +
pair A2=(0.575,0.04); pair B2=(0.03,0.816);
 
dot(O);
 
dot(O);
 
dot(X);
 
dot(X);
 
dot(Y); dot(A); dot(B);
 
dot(Y); dot(A); dot(B);
 +
dot(A1); dot(B1);
 +
dot(A2); dot(B2);
 
draw(X--O--Y);
 
draw(X--O--Y);
 
draw(A--B);
 
draw(A--B);
label("$B'$", B, W);
+
draw(A1--B1);
label("$A'$", A, S);
+
draw(A--A2);
 +
draw(B1--B2);
 +
label("$B$", B, W);
 +
label("$A$", A, S);
 +
label("$B_1$", B1, SW);
 +
label("$A_1$", A1, S);
 +
label("$B_2$", B2, E);
 +
label("$A_2$", A2, NE);
 
label("$O$", O, SW);
 
label("$O$", O, SW);
pair C=(1/8,3*sqrt(3)/8);
+
pair C=(0.18,0.56);
 +
label("$C$", C, E);
 
dot(C);
 
dot(C);
pair D=(1/8,0); dot(D);
 
pair E=(0,3*sqrt(3)/8); dot(E);
 
label("$C$", C, NE); label("$D$", D, S); label("$E$", E, W);
 
draw(D--C--E);
 
 
</asy>
 
</asy>
  
Let <math>C = (\tfrac18,\tfrac{3\sqrt3}8)</math>. <s>this is sus, furaken randomly guessed C and proceeded to prove it works</s> Draw a line through <math>C</math> intersecting the <math>x</math>-axis at <math>A'</math> and the <math>y</math>-axis at <math>B'</math>. We shall show that <math>A'B' \ge 1</math>, and that equality only holds when <math>A'=A</math> and <math>B'=B</math>.
+
Let's move a little bit from <math>A</math> to <math>A_1</math>, then <math>B</math> must move to <math>B_1</math> to keep <math>A_1B_1 = 1</math>. <math>AB</math> intersects with <math>A_1B_1</math> at <math>C</math>. Pick points <math>A_2</math> and <math>B_2</math> on <math>CA_1</math> and <math>CB</math> such that <math>CA_2 = CA</math>, <math>CB_2 = CB_1</math>, we have <math>A_1A_2 = BB_2</math>. Since <math>AA_1</math> is very small, <math>\angle CA_1A \approx 60^\circ</math>, <math>\angle CBB_1 \approx 30^\circ</math>, so <math>AA_2\approx \sqrt{3}A_1A_2</math>, <math>B_1B_2 \approx \frac{1}{\sqrt{3}}BB_2</math>, by similarity, <math>\frac{CA}{CB} \approx \frac{CA}{CB_2} = \frac{AA_2}{B_1B_2} = \frac{\sqrt{3}A_1A_2}{\frac{1}{\sqrt{3}}BB_2} = 3</math>. So the coordinates of <math>C</math> is <math>\left(\frac{1}{8}, \frac{3\sqrt{3}}{8}\right)</math>.
  
Let <math>\theta = \angle OA'C</math>. Draw <math>CD</math> perpendicular to the <math>x</math>-axis and <math>CE</math> perpendicular to the <math>y</math>-axis as shown in the diagram. Then
+
so <math>OC^2 = \frac{1}{64} + \frac{27}{64} = \frac{7}{16}</math>, the answer is <math>\boxed{023}</math>.
<cmath>8A'B' = 8CA' + 8CB' = \frac{3\sqrt3}{\sin\theta} + \frac{1}{\cos\theta}</cmath>
 
By some inequality (i forgor its name),
 
<cmath>\left(\frac{3\sqrt3}{\sin\theta} + \frac{1}{\cos\theta}\right) \cdot \left(\frac{3\sqrt3}{\sin\theta} + \frac{1}{\cos\theta}\right) \cdot (\sin^2\theta + \cos^2\theta) \ge (3+1)^3 = 64</cmath>
 
We know that <math>\sin^2\theta + \cos^2\theta = 1</math>. Thus <math>\tfrac{3\sqrt3}{\sin\theta} + \tfrac{1}{\cos\theta} \ge 8</math>. Equality holds if and only if
 
<cmath>\frac{3\sqrt3}{\sin\theta} : \frac{1}{\cos\theta} = \frac{3\sqrt3}{\sin\theta} : \frac{1}{\cos\theta} = \sin^2\theta : \cos^2\theta</cmath>
 
which occurs when <math>\theta=\tfrac\pi3</math>. Guess what, <math>\angle OAB</math> ''happens'' to be <math>\tfrac\pi3</math>, thus <math>A'=A</math> and <math>B'=B</math>. Thus, <math>AB</math> is the only segment in <math>\mathcal{F}</math> that passes through <math>C</math>. Finally, we calculate <math>OC^2 = \tfrac1{64} + \tfrac{27}{64} = \tfrac7{16}</math>, and the answer is <math>\boxed{023}</math>.
 
~Furaken
 
  
==Solution 2==
 
  
<math>y=-\tan \theta x+\sin \theta=-\sqrt{3}x+\frac{\sqrt{3}}{2}, x=\frac{\sqrt{3}-2\sin \theta}{2\sqrt{3}-2\tan \theta}</math>
+
==Video Solution==
 +
 
 +
https://youtu.be/914687Yv6SY?si=tc6XfoOIHu0gu6AL
 +
 
 +
(no calculus)
 +
 
 +
~MathProblemSolvingSkills.com
 +
 
 +
 
 +
==Video Solution==
 +
 
 +
https://youtu.be/QwLBBzHFPNE
 +
 
 +
~Steven Chen (Professor Chen Education Palace, www.professorchenedu.com)
 +
 
 +
==Query==
 +
<asy>
 +
pair O=(0,0);
 +
pair X=(1,0);
 +
pair Y=(0,1);
 +
pair A=(0.5,0); pair B=(0,sin(pi/3));
 +
dot(O);
 +
dot(X);
 +
dot(Y); dot(A); dot(B);
 +
draw(X--O--Y);
 +
draw(A--B);
 +
label("$B$", B, W);
 +
pair P=(0.5, sin(pi/3));
 +
dot(P);
 +
draw(A--P--B);
 +
label("$A$", A, S);
 +
label("$O$", O, SW);
 +
pair C=(1/8,3*sqrt(3)/8);
 +
dot(C);
 +
label("$C$", C, SW);
 +
draw(C--P); label("$P$", P, NE);
 +
</asy>
 +
Let <math>C</math> be a fixed point in the first quadrant. Let <math>A</math> be a point on the positive <math>x</math>-axis and <math>B</math> be a point on the positive <math>y</math>-axis such that <math>AB</math> passes through <math>C</math> and the length of <math>AB</math> is minimal. Let <math>P</math> be the point such that <math>OAPB</math> is a rectangle. Prove that <math>PC \perp AB</math>. (One can solve this through algebra/calculus bash, but I'm trying to find a solution that mainly uses geometry. If you know such a solution, write it here on this wiki page.) ~Furaken
 +
 
 +
I think there is such a geometry way:
 +
Let <math>DE</math> pass through <math>C</math> while point <math>D</math> is on the outside of line segment <math>OA</math> and point <math>E</math> is in between <math>O</math> and <math>B</math>. We aim to show <math>DE</math> is longer than <math>AB</math>. Now since <math>PC</math> is the altitude of triangle <math>PAB</math> yet just a cevian on the base <math>DE</math> of triangle <math>PDE</math> (thus making the height shorter than <math>PC</math>), it suffices to show the area of triangle <math>PDE</math> is bigger than that of triangle <math>PAB</math>. To do this, we compare these two triangles (let <math>DE</math> intersect <math>PA</math> at point <math>F</math>), and we just want to show <math>PF*AD > AF*AO</math>. This is trivial by similarity ratios. ~gougutheorem
 +
 
 +
Thanks! Now we know that it's possible to solve the AIME problem with only geometry. ~Furaken
  
When <math>\theta=\frac{\pi}{3}</math>, the limit of <math>x=\frac{1}{8}\implies y=\frac{3\sqrt{3}}{8}\implies \frac{7}{16}=OC^2, \boxed{023}</math>
+
==See also==
 +
{{AIME box|year=2024|num-b=11|num-a=13|n=II}}
  
~Bluesoul
+
[[Category:Intermediate Geometry Problems]]
 +
{{MAA Notice}}

Latest revision as of 11:34, 14 May 2024

Problem

Let \(O=(0,0)\), \(A=\left(\tfrac{1}{2},0\right)\), and \(B=\left(0,\tfrac{\sqrt{3}}{2}\right)\) be points in the coordinate plane. Let \(\mathcal{F}\) be the family of segments \(\overline{PQ}\) of unit length lying in the first quadrant with \(P\) on the \(x\)-axis and \(Q\) on the \(y\)-axis. There is a unique point \(C\) on \(\overline{AB}\), distinct from \(A\) and \(B\), that does not belong to any segment from \(\mathcal{F}\) other than \(\overline{AB}\). Then \(OC^2=\tfrac{p}{q}\), where \(p\) and \(q\) are relatively prime positive integers. Find \(p+q\).


Solution 1 (completely no calculus required)

Begin by finding the equation of the line $\overline{AB}$: $y= -\sqrt{3}x+\frac{\sqrt{3}}{2}$ Now, consider the general equation of all lines that belong to $\mathcal{F}$. Let $P$ be located at $(a,0)$ and $Q$ be located at $(0,b)$. With these assumptions, we may arrive at the equation $ay +bx =ab$. However, a critical condition that must be satisfied by our parameters is that $a^2+b^2=1$, since the length of $\overline{PQ}=1$.

Here's the golden trick that resolves the problem: we wish to find some point $C$ along $\overline{AB}$ such that $\overline{PQ}$ passes through $C$ iff $a=\frac{1}{2}$. It's not hard to convince oneself of this, since the property $a^2+b^2=1$ implies that if $a=\frac{1}{2}$, then $\overline{PQ}=\overline{AB}$.

We should now try to relate the point $C$ to some value of $a$. This is accomplished by finding the intersection of two lines: \[     a(-\sqrt{3}x +\frac{\sqrt{3}}{2}) + x\sqrt{1-a^2} = a\sqrt{1-a^2} \]

Where we have also used the fact that $b=\sqrt{1-a^2}$, which follows nicely from $a^2+b^2 =1$. \[     a(-\sqrt{3}x +\frac{\sqrt{3}}{2})   = (a-x)\sqrt{1-a^2} \]

Square both sides and go through some algebraic manipulations to arrive at \[     -a^4 +2xa^3+(-4x^2+3x+\frac{1}{4})a^2-2xa+x^2=0 \]

Note how $a=\frac{1}{2}$ is a solution to this polynomial, and it is logically so. If we found the set of intersections consisting of line segment $\overline{AB}$ with an identical copy of itself, every single point on the line (all $x$ values) should satisfy the equation. Thus, we can perform polynomial division to eliminate the extraneous solution $a=\frac{1}{2}$. \[     -a^3 + (2x-\frac{1}{2})a^2+(-4x^2+4x)a-2x^2=0 \]

Remember our original goal. It was to find an $x$ value such that $a=\frac{1}{2}$ is the only valid solution. Therefore, we can actually plug in $a=\frac{1}{2}$ back into the equation to look for values of $x$ such that the relation is satisfied, then eliminate undesirable answers. \[     16x^2-10x+1=0 \] This is easily factored, allowing us to determine that $x=\frac{1}{8},\frac{1}{2}$. The latter root is not our answer, since on line $\overline{AB}$, $y(\frac{1}{2})=0$, the horizontal line segment running from $(0,0)$ to $(1,0)$ covers that point. From this, we see that $x=\frac{1}{8}$ is the only possible candidate.

Going back to line $\overline{AB}, y= -\sqrt{3}x+\frac{\sqrt{3}}{2}$, plugging in $x=\frac{1}{8}$ yields $y=\frac{3\sqrt{3}}{8}$. The distance from the origin is then given by $\sqrt{\frac{1}{8^2}+(\frac{3\sqrt{3}}{8})^2} =\sqrt{\frac{7}{16}}$. That number squared is $\frac{7}{16}$, so the answer is $\boxed{023}$.


~Installhelp_hex

Solution 2

$y=-(\tan \theta) x+\sin \theta=-\sqrt{3}x+\frac{\sqrt{3}}{2}, x=\frac{\sqrt{3}-2\sin \theta}{2\sqrt{3}-2\tan \theta}$

Now, we want to find $\lim_{\theta\to\frac{\pi}{3}}\frac{\sqrt{3}-2\sin \theta}{2\sqrt{3}-2\tan \theta}$. By L'Hôpital's rule, we get $\lim_{\theta\to\frac{\pi}{3}}\frac{\sqrt{3}-2\sin \theta}{2\sqrt{3}-2\tan \theta}=\lim_{\theta\to\frac{\pi}{3}}cos^3{x}=\frac{1}{8}$. This means that $y=\frac{3\sqrt{3}}{8}\implies OC^2=\frac{7}{16}$, so we get $\boxed{023}$.

~Bluesoul

Solution 3

The equation of line $AB$ is \[ y = \frac{\sqrt{3}}{2} x - \sqrt{3} x.  \hspace{1cm} (1) \]

The position of line $PQ$ can be characterized by $\angle QPO$, denoted as $\theta$. Thus, the equation of line $PQ$ is

\[ y = \sin \theta - \tan \theta \cdot x . \hspace{1cm} (2) \]

Solving (1) and (2), the $x$-coordinate of the intersecting point of lines $AB$ and $PQ$ satisfies the following equation:

\[ \frac{\frac{\sqrt{3}}{2} - \sqrt{3} x}{\sin \theta} + \frac{x}{\cos \theta} = 1 . \hspace{1cm} (1) \]

We denote the L.H.S. as $f \left( \theta; x \right)$.

We observe that $f \left( 60^\circ ; x \right) = 1$ for all $x$. Therefore, the point $C$ that this problem asks us to find can be equivalently stated in the following way:

We interpret Equation (1) as a parameterized equation that $x$ is a tuning parameter and $\theta$ is a variable that shall be solved and expressed in terms of $x$. In Equation (1), there exists a unique $x \in \left( 0, 1 \right)$, denoted as $x_C$ ($x$-coordinate of point $C$), such that the only solution is $\theta = 60^\circ$. For all other $x \in \left( 0, 1 \right) \backslash \{ x_C \}$, there are more than one solutions with one solution $\theta = 60^\circ$ and at least another solution.

Given that function $f \left( \theta ; x \right)$ is differentiable, the above condition is equivalent to the first-order-condition \[ \frac{\partial f \left( \theta ; x_C \right) }{\partial \theta} \bigg|_{\theta = 60^\circ} = 0 . \]

Calculating derivatives in this equation, we get \[ - \left( \frac{\sqrt{3}}{2} - \sqrt{3} x_C \right) \frac{\cos 60^\circ}{\sin^2 60^\circ} + x_C \frac{\sin 60^\circ}{\cos^2 60^\circ} = 0. \]

By solving this equation, we get \[ x_C = \frac{1}{8} . \]

Plugging this into Equation (1), we get the $y$-coordinate of point $C$: \[ y_C = \frac{3 \sqrt{3}}{8} . \]

Therefore, \begin{align*} OC^2 & = x_C^2 + y_C^2 \\ & = \frac{7}{16} . \end{align*}

Therefore, the answer is $7 + 16 = \boxed{\textbf{(23) }}$.

~Steven Chen (Professor Chen Education Palace, www.professorchenedu.com)

Solution 4 (coordinate bash)

Let $s$ be a segment in $\mathcal{F}$ with x-intercept $a$ and y-intercept $b$. We can write $s$ as \begin{align*} \frac{x}{a} + \frac{y}{b} &= 1 \\ y &= b(1 - \frac{x}{a}). \end{align*} Let the unique point in the first quadrant $(x, y)$ lie on $s$ and no other segment in $\mathcal{F}$. We can find $x$ by solving \[b(1 - \frac{x}{a}) = (b + db)(1 - \frac{x}{a + da})\] and taking the limit as $da, db \to 0$. Since $s$ has length $1$, $a^2 + b^2 = 1^2$ by the Pythagorean theorem. Solving this for $db$, we get \begin{align*} a^2 + b^2 &= 1 \\ b^2 &= 1 - a^2 \\ \frac{db^2}{da} &= \frac{d(1 - a^2)}{da} \\ 2a\frac{db}{da} &= -2a \\ db &= -\frac{a}{b}da. \end{align*} After we substitute $db = -\frac{a}{b}da$, the equation for $x$ becomes \[b(1 - \frac{x}{a}) = (b -\frac{a}{b} da)(1 - \frac{x}{a + da}).\]

In $\overline{AB}$, $a = \frac{1}{2}$ and $b = \frac{\sqrt{3}}{2}$. To find the x-coordinate of $C$, we substitute these into the equation for $x$ and get \begin{align*} \frac{\sqrt{3}}{2}(1 - \frac{x}{\frac{1}{2}}) &= (\frac{\sqrt{3}}{2} - \frac{\frac{1}{2}}{\frac{\sqrt{3}}{2}} da)(1 - \frac{x}{\frac{1}{2} + da}) \\ \frac{\sqrt{3}}{2}(1 - 2x) &= (\frac{\sqrt{3}}{2} - \frac{da}{\sqrt{3}})(1 - \frac{x}{\frac{1 + 2da}{2}}) \\ \frac{\sqrt{3}}{2} - \sqrt{3}x &= \frac{3 - 2da}{2\sqrt{3}}(1 - \frac{2x}{1 + 2da}) \\ \frac{\sqrt{3}}{2} - \sqrt{3}x &= \frac{3 - 2da}{2\sqrt{3}} \cdot \frac{1 + 2da - 2x}{1 + 2da} \\ \frac{\sqrt{3}}{2} - \sqrt{3}x &= \frac{3 + 6da - 6x - 2da - 4da^2 + 4xda}{2\sqrt{3} + 4\sqrt{3}da} \\ (\frac{\sqrt{3}}{2} - \sqrt{3}x)(2\sqrt{3} + 4\sqrt{3}da) &= 3 + 6da - 6x - 2da - 4da^2 + 4xda \\ 3 + 6da - 6x - 12xda &= 3 + 4da - 6x - 4da^2 + 4xda \\ 2da &= -4da^2 + 16xda \\ 16xda &= 2da + 4da^2 \\ x &= \frac{da + 2da^2}{8da}. \end{align*} We take the limit as $da \to 0$ to get \[x = \lim_{da \to 0} \frac{da + 2da^2}{8da} = \lim_{da \to 0} \frac{1 + 2da}{8} = \frac{1}{8}.\] We substitute $x = \frac{1}{8}$ into the equation for $\overline{AB}$ to find the y-coordinate of $C$: \[y = b(1 - \frac{x}{a}) = \frac{\sqrt{3}}{2}(1 - \frac{\frac{1}{8}}{\frac{1}{2}}) = \frac{3\sqrt{3}}{8}.\] The problem asks for \[OC^2 = x^2 + y^2 = (\frac{1}{8})^2 + (\frac{3\sqrt{3}}{8})^2 = \frac{7}{16} = \frac{p}{q},\] so $p + q = 7 + 16 = \boxed{023}$.

Solution 5 (small perturb)

[asy] pair O=(0,0); pair X=(1,0); pair Y=(0,1); pair A=(0.5,0); pair B=(0,sin(pi/3)); pair A1=(0.6,0); pair B1=(0,0.8); pair A2=(0.575,0.04); pair B2=(0.03,0.816); dot(O); dot(X); dot(Y); dot(A); dot(B); dot(A1); dot(B1); dot(A2); dot(B2); draw(X--O--Y); draw(A--B); draw(A1--B1); draw(A--A2); draw(B1--B2); label("$B$", B, W); label("$A$", A, S); label("$B_1$", B1, SW); label("$A_1$", A1, S); label("$B_2$", B2, E); label("$A_2$", A2, NE); label("$O$", O, SW); pair C=(0.18,0.56); label("$C$", C, E); dot(C); [/asy]

Let's move a little bit from $A$ to $A_1$, then $B$ must move to $B_1$ to keep $A_1B_1 = 1$. $AB$ intersects with $A_1B_1$ at $C$. Pick points $A_2$ and $B_2$ on $CA_1$ and $CB$ such that $CA_2 = CA$, $CB_2 = CB_1$, we have $A_1A_2 = BB_2$. Since $AA_1$ is very small, $\angle CA_1A \approx 60^\circ$, $\angle CBB_1 \approx 30^\circ$, so $AA_2\approx \sqrt{3}A_1A_2$, $B_1B_2 \approx \frac{1}{\sqrt{3}}BB_2$, by similarity, $\frac{CA}{CB} \approx \frac{CA}{CB_2} = \frac{AA_2}{B_1B_2} = \frac{\sqrt{3}A_1A_2}{\frac{1}{\sqrt{3}}BB_2} = 3$. So the coordinates of $C$ is $\left(\frac{1}{8}, \frac{3\sqrt{3}}{8}\right)$.

so $OC^2 = \frac{1}{64} + \frac{27}{64} = \frac{7}{16}$, the answer is $\boxed{023}$.


Video Solution

https://youtu.be/914687Yv6SY?si=tc6XfoOIHu0gu6AL

(no calculus)

~MathProblemSolvingSkills.com


Video Solution

https://youtu.be/QwLBBzHFPNE

~Steven Chen (Professor Chen Education Palace, www.professorchenedu.com)

Query

[asy] pair O=(0,0); pair X=(1,0); pair Y=(0,1); pair A=(0.5,0); pair B=(0,sin(pi/3)); dot(O); dot(X); dot(Y); dot(A); dot(B); draw(X--O--Y); draw(A--B); label("$B$", B, W); pair P=(0.5, sin(pi/3)); dot(P); draw(A--P--B); label("$A$", A, S); label("$O$", O, SW); pair C=(1/8,3*sqrt(3)/8); dot(C); label("$C$", C, SW); draw(C--P); label("$P$", P, NE); [/asy] Let $C$ be a fixed point in the first quadrant. Let $A$ be a point on the positive $x$-axis and $B$ be a point on the positive $y$-axis such that $AB$ passes through $C$ and the length of $AB$ is minimal. Let $P$ be the point such that $OAPB$ is a rectangle. Prove that $PC \perp AB$. (One can solve this through algebra/calculus bash, but I'm trying to find a solution that mainly uses geometry. If you know such a solution, write it here on this wiki page.) ~Furaken

I think there is such a geometry way: Let $DE$ pass through $C$ while point $D$ is on the outside of line segment $OA$ and point $E$ is in between $O$ and $B$. We aim to show $DE$ is longer than $AB$. Now since $PC$ is the altitude of triangle $PAB$ yet just a cevian on the base $DE$ of triangle $PDE$ (thus making the height shorter than $PC$), it suffices to show the area of triangle $PDE$ is bigger than that of triangle $PAB$. To do this, we compare these two triangles (let $DE$ intersect $PA$ at point $F$), and we just want to show $PF*AD > AF*AO$. This is trivial by similarity ratios. ~gougutheorem

Thanks! Now we know that it's possible to solve the AIME problem with only geometry. ~Furaken

See also

2024 AIME II (ProblemsAnswer KeyResources)
Preceded by
Problem 11
Followed by
Problem 13
1 2 3 4 5 6 7 8 9 10 11 12 13 14 15
All AIME Problems and Solutions

The problems on this page are copyrighted by the Mathematical Association of America's American Mathematics Competitions. AMC logo.png